You are on page 1of 31

Page 1 of 31

SFG 2022 | LEVEL 1 | Test #12 – Solutions |


Q.1) With reference to the ‘Fiscal marksmanship’, which of the following statements is correct?
a) It is the difference between the budgetary expenditures and the government’s revenue receipts.
b) It refers to the accuracy of the government’s forecast of fiscal parameters.
c) It refers to the sharing of tax revenues by the Union government with the states as per the spirit of fiscal
federalism.
d) It is the deflationary effect of a progressive taxation system on a country's economy.

Ans) b
Exp) Option b is correct.
Fiscal marksmanship essentially refers to the accuracy of the government’s forecast of fiscal parameters such as
revenues, expenditures and deficits etc. In other words, if the difference between what the government projected
in the Budget and the actual figures a year later is large then it reflects poor fiscal marksmanship. It helps
determine the creditability of the budget numbers of the government.
Source: https://indianexpress.com/article/explained/explained-what-ails-with-the-credibility-of-indias-
budget-numbers-6239577/

Q.2) Consider the following pairs:


Types of Deficit Meaning
1. Effective Excess of government’s
Revenue revenue expenditure over
Deficit total receipts.
2. Primary Deficit Indicates the government's
borrowings that are utilised
to pay the interest on loans.
3. Monetised Part of public deficit
Deficit financed by borrowings
from abroad.

Which of the pairs given above is/are correctly matched?


a) 1 and 2 only
b) 2 only
c) 2 and 3 only
d) 1 and 3 only

Ans) b
Exp) Option b is correct.
Pair 1 is incorrect. The revenue deficit refers to the excess of government’s revenue expenditure over revenue
receipts (not total receipts).
Revenue deficit = Revenue expenditure – Revenue receipts
On the contrary, Effective Revenue Deficit is the difference between revenue deficit and grants for creation of
capital assets. The concept of effective revenue deficit was suggested by the Rangarajan Committee on Public
Expenditure and introduced in 2011-12 budget. It is aimed to deduct the money used out of borrowing to finance
capital expenditure. The concept has been introduced to ascertain the actual deficit in the revenue account after
adjusting for expenditure of capital nature.
Pair 2 is correct. Primary deficit is the difference between the fiscal deficit of the current year and the interest
paid by the government on loans obtained in the past. It indicates the government's borrowings that are utilised
to pay the interest on loans rather than on capital expenditure.
Pair 3 is incorrect. Monetized deficit is the part of the government deficit which is financed by borrowing from
RBI. Monetised deficit, also known as debt monetisation, is the monetary support that Reserve Bank of India (RBI)
extends to the Centre as part of the government's borrowing programme. In other words, the term refers to the
purchase of government bonds by the central bank to finance the spending needs of the government.

Forum Learning Centre: Delhi - 2nd Floor, IAPL House, 19 Pusa Road, Karol Bagh, New Delhi - 110005 | Patna - 2nd floor, AG Palace, E Boring Canal Road,
Patna, Bihar 800001 | Hyderabad - 1st & 2nd Floor, SM Plaza, RTC X Rd, Indira Park Road, Jawahar Nagar, Hyderabad, Telangana 500020
9821711605 | https://academy.forumias.com | admissions@forumias.academy | helpdesk@forumias.academy
Page 2 of 31

SFG 2022 | LEVEL 1 | Test #12 – Solutions |


Monetisation of deficit was in practice in India till 1997, whereby the central bank automatically monetised
government deficit through the issuance of ad-hoc treasury bills.
Source: Macroeconomics, NCERT XII, Chapter-5, Pg. 65
http://arthapedia.in/index.php?title=Effective_Revenue_deficit
https://economictimes.indiatimes.com/markets/stocks/news/deficit-monetisation-is-it-really-as-simple-
as-rbi-printing-more-money/articleshow/75968757.cms?from=mdr

Q.3) If another global financial crisis happens in the near future, which of the following action/policies are most
likely to give some immunity to India?
1. Not depending on short-term foreign borrowings
2. Opening up to more foreign banks
3. Maintaining full capital account convertibility
Select the correct answer using the code given below:
a) 1 only
b) 1 and 2 only
c) 3 only
d) 1, 2 and 3

Ans) a
Exp) Option a is correct.
Statement 1 is correct. The Short-term debts have to be returned at a shorter interval. This renders an economy
vulnerable if the economy is already facing economic crisis as it has an obligation to return the debt as well as
interest payments (debt service). Example: Many economies like Mexico, Argentina etc who faced crisis were
seen to have borrowed large amounts of short-maturity debt. So, the ideal scenarios is not to depend upon short-
term debt during crisis.
Statement 2 is incorrect. Opening up to the foreign banks and depending upon them is not a good idea during
crisis. It has been empirically observed in World Bank Research that in many cases (like during Global Financial
Crisis 2007-08) that foreign banks reduced their lending during crisis. Many of them choose to retreat from
cross-border banking in general, including through cutting back on new entry.
Statement 3 is incorrect. Capital account convertibility would mean that there is no restriction on conversion of
the domestic currency into a foreign currency. It is recognised that capital flows are sensitive to macroeconomic
conditions. Any deterioration in fiscal conditions, inflation management, balance of payments, or any other
economic crisis may cause a cessation or reversal of capital flows. This might make the economy vulnerable if it
already suffering from economic crisis.
Source) UPSC 2020

Q.4) Regarding the transfer payments, which of the following statements in incorrect?
a) It is the one-way payment of money made by the government to the people.
b) These payments are aimed at redistribution of income.
c) These payments are made without exchange of any goods or services.
d) Subsidies paid to exporters and farmers are considered as transfer payments.

Ans) d
Exp) Option d is correct.
Option a is correct. Transfer payment is the payment by the government in grants, allowances, pensions etc to
people such as pensioners, widows, sick etc.
Transfer payments are a one-way payment of money for which no good or service is received in exchange.
Option b is correct. Transfer payment is aimed at redistribution of income. Government use such payments as
means of income redistribution under social welfare programs. It includes payments such as social security, old
age or disability pensions, student grants, unemployment compensation, and so on. There is a need to
differentiate them from subsidies. Transfer payments are a part of personal income.

Forum Learning Centre: Delhi - 2nd Floor, IAPL House, 19 Pusa Road, Karol Bagh, New Delhi - 110005 | Patna - 2nd floor, AG Palace, E Boring Canal Road,
Patna, Bihar 800001 | Hyderabad - 1st & 2nd Floor, SM Plaza, RTC X Rd, Indira Park Road, Jawahar Nagar, Hyderabad, Telangana 500020
9821711605 | https://academy.forumias.com | admissions@forumias.academy | helpdesk@forumias.academy
Page 3 of 31

SFG 2022 | LEVEL 1 | Test #12 – Solutions |


Option c is correct. Transfer payment doesn’t involve a return of any productive service from the beneficiaries
to the government. These payments are considered to be exhaustive because they do not directly absorb
resources or create output. This implies that the transfer is made without any exchange of goods or services.
Examples of certain transfer payments include welfare (financial aid), social security, and government making
subsidies for certain businesses.
Option d is incorrect. Subsidies paid to exporters, farmers, manufacturers are not considered transfer payments
because they are linked to a commodity transaction.
Source: https://www.indianeconomy.net/glossary/transfer-payments/

Q.5) In the context of Indian economy, which of the following is/are included within the scope of Gross Fixed
Capital Formation (GFCF)?
1. Acquisition of produced assets
2. Acquisition of defence equipment
3. Purchases of second-hand assets by producers
Select the correct answer using the code given below:
a) 1 only
b) 1 and 3 only
c) 1 and 2 only
d) 2 and 3 only

Ans) b
Exp) Option b is correct.
Statement 1 is correct. Gross capital formation refers to the ‘aggregate of gross additions to fixed assets (that is
fixed capital formation) plus change in stocks of inventories’ during the counting period. Fixed asset refers to the
construction, machinery and equipment. Gross fixed capital formation (GFCF) is defined as the acquisition of
produced assets (including purchases of second-hand assets), including the production of such assets by
producers for their own use.
Statement 2 is incorrect. Construction for military purposes (other than construction or alteration of family
dwellings for military personnel), acquisition of defence equipment, durable goods in the hands of the households
and increase in the stocks of defence materials are excluded from the scope of gross fixed capital formation.
Statement 3 is correct. Gross fixed capital formation (GFCF) includes the acquisition of produced assets including
purchases of second-hand assets. It also includes the production of such assets by producers for their own use,
minus disposals. However, the expenditure incurred on usual/ routine repair and maintenance is not covered
for compilation of capital formation.
Source:
http://mospi.nic.in/sites/default/files/reports_and_publication/statistical_manual/Chapter%2025.pdf

Q.6) There has been a persistent deficit budget year after year. Which of the following actions can be taken by
the government to reduce the deficit?
1. Reducing revenue expenditure
2. Introducing new welfare schemes
3. Rationalizing subsidies
4. Expanding industries
Select the correct answer using the code given below.
a) 1 and 3 only
b) 2 and 3 only
c) 1 only
d) 1, 2, 3 and 4

Ans) a
Exp) Option a is correct

Forum Learning Centre: Delhi - 2nd Floor, IAPL House, 19 Pusa Road, Karol Bagh, New Delhi - 110005 | Patna - 2nd floor, AG Palace, E Boring Canal Road,
Patna, Bihar 800001 | Hyderabad - 1st & 2nd Floor, SM Plaza, RTC X Rd, Indira Park Road, Jawahar Nagar, Hyderabad, Telangana 500020
9821711605 | https://academy.forumias.com | admissions@forumias.academy | helpdesk@forumias.academy
Page 4 of 31

SFG 2022 | LEVEL 1 | Test #12 – Solutions |


Option a is correct: A budget deficit is when spending exceeds income. The term applies to governments,
although individuals, companies, and other organizations can run deficits.
Introducing new welfare schemes will further inflate the budget. Expanding industries by budgetary support will
not add anything in the short-run to the tax revenues of the government and thus will increase the budget deficit.
Rationalizing subsidies and reducing revenue expenditure are two direct ways of reducing the fiscal burden of
the government of India.
Source) UPSC 2015

Q.7) With reference to the Medium-Term Fiscal Policy Statement, consider the following statements:
1. It sets three-year rolling target for specific fiscal indicators in relation to GDP.
2. It is presented as part of General Budget by the Union government.
3. It is mandated by the Fiscal Responsibility and Budget Management Act, 2003.
Which of the statements given above is/are correct?
a) 1 and 2 only
b) 2 and 3 only
c) 1 and 3 only
d) 1, 2 and 3

Ans) d
Exp) Option d is correct.
Statement 1 is correct. The Medium-term Fiscal Policy Statement sets a three-year rolling target for six specific
fiscal indicators in relation to gross domestic product (GDP) at market prices:
1) Revenue Deficit
2) Fiscal Deficit
3) Tax Revenue
4) Primary Deficit
5) Non-Tax revenue
6) Central Government Debt
Statement 2 is correct. The Medium-term Fiscal Policy Statement is presented as part of the General Budget. It
is presented to Parliament under Section 3(2) of the Fiscal Responsibility and Budget Management (FRBM) Act,
2003.
Statement 3 is correct. The Macro-Economic Framework Statement is presented to Parliament as mandated by
the Fiscal Responsibility and Budget Management Act, 2003. It makes an assessment of the growth prospects of
the economy with specific underlying assumptions. On the contrary, Medium-term Fiscal Policy Statement
examines whether revenue expenditure can be financed through revenue receipts on a sustainable basis and how
productively capital receipts including market borrowings are being utilised.
Knowledge Base:
Apart from Medium-term Fiscal Policy Statement, two other statements are presented as part of FRBM Act, 2003:
1) The Fiscal Policy Strategy Statement sets the priorities of the government in the fiscal area, examining current
policies and justifying any deviation in important fiscal measures.
2) The Macroeconomic Framework Statement assesses the prospects of the economy with respect to the GDP
growth rate, fiscal balance of the central government and external balance.
Source: Macroeconomics, NCERT XII, Chapter-5, Pg. 64
https://economictimes.indiatimes.com/markets/stocks/news/what-is-medium-term-fiscal-policy-
statement/articleshow/56737837.cms?from=mdr
https://www.business-standard.com/about/what-is-macro-economic-framework-statement
https://www.indiabudget.gov.in/doc/Key_to_Budget_Document_2021.pdf

Q.8) Which of the following are the primary objectives of fiscal policy in India?
1. Maintain economy’s growth rate.
2. Ensure liquidity in the market
3. Maintain price stability

Forum Learning Centre: Delhi - 2nd Floor, IAPL House, 19 Pusa Road, Karol Bagh, New Delhi - 110005 | Patna - 2nd floor, AG Palace, E Boring Canal Road,
Patna, Bihar 800001 | Hyderabad - 1st & 2nd Floor, SM Plaza, RTC X Rd, Indira Park Road, Jawahar Nagar, Hyderabad, Telangana 500020
9821711605 | https://academy.forumias.com | admissions@forumias.academy | helpdesk@forumias.academy
Page 5 of 31

SFG 2022 | LEVEL 1 | Test #12 – Solutions |


4. Ensure proper functioning of financial intermediaries
5. Achievement of full employment
Select the correct answer using the code given below:
a) 1 and 5 only
b) 1, 3 and 5 only
c) 2, 3 and 4 only
d) 1, 2, 3, 4 and 5

Ans) b
Exp) Option b is correct
Fiscal policy in India is the guiding force that helps the government decide how much money it should spend to
support the economic activity, and how much revenue it must earn from the system, to keep the wheels of the
economy running smoothly. Through the fiscal policy, the government of a country controls the flow of tax
revenues and public expenditure to navigate the economy.
Option b is correct. The main objectives of fiscal policy in India are:
1) Maintaining the economy’s growth rate so that certain economic goals can be achieved.
2) Ensuring price stability by controlling the price level of the country so that when the inflation is too high,
prices can be regulated.
3) Achievement of full employment, or near full employment, as a tool to recover from low economic activity.
Ensuring liquidity in the market is the objective of monetary policy while financial intermediaries are regulated
under various acts.
Source: https://www.financialexpress.com/what-is/fiscal-policy-meaning/1771755/

Q.9) Which of the following is/are included in the capital budget of the Government of India?
1. Expenditure on acquisition of assets like roads, buildings, machinery, etc.
2. Loans received from foreign governments
3. Loans and advances granted to the States and Union Territories
Select the correct answer using the code given below.
a) 1 only
b) 2 and 3 only
c) 1 and 3 only
d) 1, 2 and 3

Ans) d
Exp) Option d is correct.
Statements 2 is correct. Capital Budget consists of capital receipts and payments. It also incorporates
transactions in the Public Account. Capital receipts includes borrowings by the government from the Reserve
Bank and other parties through sale of treasury bills, loans received from foreign bodies and governments, and
recoveries of loans granted by the Central government to State and Union Territories (UTs) and other parties.
Statement 1 and 3 are correct. Capital payments consist of capital expenditure on acquisition of assets like land,
buildings, machinery, and equipment, as also investments in shares, loans and advances granted by the Central
government to state and Union Territory governments, government companies, corporations and other parties.
Source) UPSC 2016

Q.10) Consider the following statements with respect to the Financial Stability and Development Council (FSDC)
and Goods and Services Tax (GST) Council:
1. While FSDC is a statutory body, GST council is a constitutional body.
2. While state governments are part of GST council, they have no representation in FSDC.
3.While FSDC aims at strengthening inter-regulatory coordination, GST council makes recommendations to the
Union regarding indirect taxes.
Which of the statements given above is/are correct?
a) 1 and 2 only

Forum Learning Centre: Delhi - 2nd Floor, IAPL House, 19 Pusa Road, Karol Bagh, New Delhi - 110005 | Patna - 2nd floor, AG Palace, E Boring Canal Road,
Patna, Bihar 800001 | Hyderabad - 1st & 2nd Floor, SM Plaza, RTC X Rd, Indira Park Road, Jawahar Nagar, Hyderabad, Telangana 500020
9821711605 | https://academy.forumias.com | admissions@forumias.academy | helpdesk@forumias.academy
Page 6 of 31

SFG 2022 | LEVEL 1 | Test #12 – Solutions |


b) 3 only
c) 1 and 3 only
d) 2 and 3 only

Ans) d
Exp) Option d is correct.
Statement 1 is incorrect. The Financial Stability and Development Council (FSDC) was set up by the Government
as the apex level forum in December 2010.It is not a statutory body. On the contrary, Article 279A of the
constitution enables the formation of the GST Council by the President to administer & govern GST. Thus, it is a
constitutional body.
Statement 2 is correct. The Chairperson of FSDC is Union Finance Minister of India. The members include: Heads
of all Financial Sector Regulators (RBI, SEBI, PFRDA & IRDA), Finance Secretary, Department of Economic Affairs
(DEA), Secretary, Department of Financial Services (DFS) and Chief Economic Adviser. In 2018, Government had
reconstituted FSDC to include: Minister of State responsible for the Department of Economic Affairs (DEA),
Secretary of the Department of Electronics and Information Technology, Chairperson of the Insolvency and
Bankruptcy Board of India (IBBI) and Revenue Secretary. Thus, there is no representation of state governments.
On the other hand, the Union Finance Minister of India is a Chairman of the GST Council. Ministers nominated
by the state governments are members of the GST Council.
Statement 3 is correct. FSDC aims to strengthen and institutionalize the mechanism for maintaining financial
stability, enhancing inter-regulatory coordination and promoting financial sector development. The GST council
is the key decision-making body that recommends indirect tax rate, tax exemption, the due date of forms, tax
laws, and tax deadlines, keeping in mind special rates and provisions for some states.
Knowledge Base:
The GST council is devised in such a way that the center has 1/3rd voting power and the states have 2/3rd. The
decisions are taken by the 3/4th majority. A mechanism for resolving disputes arising out of its recommendations
also decided by the Council itself.
Source: https://dea.gov.in/sites/default/files/StrucFSDC.pdf
https://blog.forumias.com/analysis-of-gst-regime-in-india/

Q.11) Consider the following statements with respect to the impact of high fiscal deficit in an economy:
1. Decrease in the real interest rates.
2. Decline in bond yields
3. May lead to crowding out of private investments.
Which of the statements given above is/are correct?
a) 1 and 2 only
b) 2 and 3 only
c) 3 only
d) 1, 2 and 3

Ans) c
Exp) Option c is correct.
Bond yield is the return an investor gets on a bond or on a particular government security. The major factors
affecting the yield is the monetary policy of the Reserve Bank of India, especially the course of interest rates, the
fiscal position of the government and its borrowing programme, global markets, economy, and inflation.
Statement 1 is incorrect. Fiscal deficits reduce national savings and increase aggregate demand. This creates an
excess supply of government debt, leading to higher real interest rates. High government borrowings also raise
the interest rate and crowd out private investment.
Statement 2 is incorrect. High fiscal deficit suggests a busy year for bond issuances and that puts an upward
pressure on bond yields. High fiscal deficit leads to an increase in market interest rates. A rising interest rates
cause bond prices to fall, and bond yields to rise.
Statement 3 is correct. High fiscal deficit, when financed through market borrowings, chokes off or ‘crowds out’
private investment. A rise in the fiscal deficit raises income, which in turn raises the demand for money. Since

Forum Learning Centre: Delhi - 2nd Floor, IAPL House, 19 Pusa Road, Karol Bagh, New Delhi - 110005 | Patna - 2nd floor, AG Palace, E Boring Canal Road,
Patna, Bihar 800001 | Hyderabad - 1st & 2nd Floor, SM Plaza, RTC X Rd, Indira Park Road, Jawahar Nagar, Hyderabad, Telangana 500020
9821711605 | https://academy.forumias.com | admissions@forumias.academy | helpdesk@forumias.academy
Page 7 of 31

SFG 2022 | LEVEL 1 | Test #12 – Solutions |


the interest rate is determined by the demand for and supply of money, an increase in demand for money
inevitably leads to an increase in interest rates. The rise in interest rate, in turn, chokes off private investment as
it raises cost for businesses. As a result, even when public spending increases, there is no net increase in
employment and output in the economy.
Source: https://timesofindia.indiatimes.com/blogs/economic-update/rising-fiscal-deficit-and-its-
economic-impact/
https://www.cbgaindia.org/blog/fiscal-deficit-bad/

Q.12) Which one of the following statements appropriately describes the “fiscal stimulus”?
a) It is a massive investment by the Government in manufacturing sector to ensure the supply of goods to meet
the demand surge caused by rapid economic growth
b) It is an intense affirmative action of the Government to boost economic activity in the country
c) It is Government’s intensive action on financial institutions to ensure disbursement of loans to agriculture and
allied sectors to promote greater food production and contain food inflation
d) It is an extreme affirmative action by the Government to pursue its policy of financial inclusion

Ans) b
Exp) Option b is correct.
Option b is correct. Fiscal stimulus is an action taken by government to increase aggregate demand in the
economy. It can be in form of tax cuts or increased public spending and Fiscal stimulus also refers to policy
measures undertaken by a government that typically reduce taxes or regulations—or increase government
spending—in order to boost economic activity.
Source) UPSC 2011

Q.13) Consider the following items:


1. Corporate Tax
2. Goods and Services Tax
3. Personal Income Tax
4. Custom Duty
5. Excise Duty
Arrange the given sources of tax collection for the Union Government in ascending order, for the year 2020-21.
a) 4-5-1-3-2
b) 4-2-3-4-1
c) 5-3-2-1-4
d) 2-1-3-4-5

Ans) a
Exp) Option a is correct.
Government's primary source of earning money is from taxes and non-tax revenues. Taxes are collected in the
form of direct and indirect ways. Direct taxes include income tax, real property tax, personal property tax, or
taxes on assets; while some of the indirect tax modes include GST, customs duty and tax deducted at source
(TDS). On the other hand, non-tax revenue is the recurring income earned by the government from sources other
than taxes. The top receipts under this are interest and dividends and profits received from public sector
companies.
Option a is correct. Direct taxes (personal income tax and corporate tax) accounted for 56.4% of total revenues
in 2020-21 with corporate tax at 28.1% and personal income tax at 28.3%. In 2020-21, 28.5% of the revenue came
from GST followed by personal income tax and corporate tax.
Source: https://timesofindia.indiatimes.com/business/faqs/budget-faqs/budget-2021-what-are-the-
income-sources-of-government/articleshow/80514295.cms

Forum Learning Centre: Delhi - 2nd Floor, IAPL House, 19 Pusa Road, Karol Bagh, New Delhi - 110005 | Patna - 2nd floor, AG Palace, E Boring Canal Road,
Patna, Bihar 800001 | Hyderabad - 1st & 2nd Floor, SM Plaza, RTC X Rd, Indira Park Road, Jawahar Nagar, Hyderabad, Telangana 500020
9821711605 | https://academy.forumias.com | admissions@forumias.academy | helpdesk@forumias.academy
Page 8 of 31

SFG 2022 | LEVEL 1 | Test #12 – Solutions |


Q.14) With reference to inverted duty structure, consider the following statements:
1. Under this, tax rate on inputs is higher than the tax rate on the final product.
2. Under Goods and Services Tax (GST) regime, the inverted duty refund is also applicable on input services.
Which of the statements given above is/are correct?
a) 1 only
b) 2 only
c) Both 1 and 2
d) Neither 1 nor 2

Ans) a
Exp) Option a is correct.
Statement 1 is correct. Under GST Regime the Inverted Duty Structure refers to the cases where the rate of tax
on inputs received are higher than the rate of tax on final good. When taxes on the final product is lower than
the taxes charged on inputs, an inverse input tax credit gets accumulated that has to be refunded by the
government in the majority of the cases.
Statement 2 is incorrect. Earlier under GST Regime the Inverted Duty Structure have wider scope as compared
to Pre-GST regime because it included input services also. But recently the Supreme Court held that inverted
duty refund is admissible only with respect to inputs and not for input services.
Source: https://www.outlookindia.com/website/story/business-news-explained-what-is-the-inverted-
duty-structure-and-why-is-the-gst-council-rectifying-it/395406
https://www.thehindubusinessline.com/economy/gst-sc-rules-on-refund-in-case-of-inverted-duty-
structure/article36426172.ece

Q.15) Why is the Government of India disinvesting its equity in the Central Public Sector Enterprises (CPSEs)?
1. The Government intends to use the revenue earned-from the disinvestment mainly to pay back the external
debt.
2. The Government no longer intends to retain the management control of the CPSEs.
Which of the statements given above is/are correct?
a) 1 only
b) 2 only
c) Both 1 and 2
d) Neither 1 nor 2

Ans) d
Exp) Option d is correct.
Statement 1 is incorrect. Disinvestment means the sale of assets by the government in the Central and state
public sector enterprises, projects, or other fixed assets. The government undertakes disinvestment to reduce
the fiscal burden on the exchequer, or to raise money for meeting specific needs, such as to bridge the revenue
shortfall from other regular sources.
Main objectives of Disinvestment in India:
1) Reducing the fiscal burden on the exchequer
2) Improving public finances
3) Encouraging private ownership
4) Funding growth and development programmes
5) Maintaining and promoting competition in the market
Statement 2 is incorrect. The government in the Budget 2021 unveiled the Disinvestment/Strategic
Disinvestment Policy. Under which it identified four sectors -- Atomic energy, Space and Defence; Transport and
Telecommunications; Power, Petroleum, Coal and other minerals; and Banking, Insurance and financial services
-- as strategic sectors, where bare minimum CPSEs would be retained. The objectives of disinvestment is as
pointed above and not Government no longer intends to retain the management control of the CPSEs.
Source) UPSC 2011

Forum Learning Centre: Delhi - 2nd Floor, IAPL House, 19 Pusa Road, Karol Bagh, New Delhi - 110005 | Patna - 2nd floor, AG Palace, E Boring Canal Road,
Patna, Bihar 800001 | Hyderabad - 1st & 2nd Floor, SM Plaza, RTC X Rd, Indira Park Road, Jawahar Nagar, Hyderabad, Telangana 500020
9821711605 | https://academy.forumias.com | admissions@forumias.academy | helpdesk@forumias.academy
Page 9 of 31

SFG 2022 | LEVEL 1 | Test #12 – Solutions |


Q.16) Which of the following is/are the components of revenue expenditure in Union budget?
1. All grants given to state governments
2. Grant given to union territories for creation of capital assets
3. Payment of subsidies
Which of the statements given above is/are correct?
a) 1 only
b) 2 and 3 only
c) 2 only
d) 1, 2 and 3

Ans) d
Exp) Option d is correct.
Revenue expenditures are the ongoing operating expenses, which are short-term expenses used to run the daily
business operations. Revenue expenses related to existing assets include repairs and regular maintenance as well
as repainting and renewal expenses. Revenue expenditures can be considered to be recurring expenses in
contrast to the one-off nature of most capital expenditures.
Option 1 and 2 are correct. The Union government’s revenue expenditure comprises money spent on revenue
account that is the amount spent on running its elaborate machinery. It includes all grants given to state
governments and Union territories. These are treated as revenue expenditure, even if some of these grants may
be used for the creation of capital assets.
Option 3 is correct. In India, the payment of subsidies is also included in revenue expenditure. The central
government pays subsidy under three major heads – food subsidy, fertilizer subsidy and fuel subsidy.
Source: https://www.financialexpress.com/what-is/revenue-expenditure-meaning/1620283/

Q.17) With reference to Indian economy, which of the following is correct regarding a ‘Balanced Budget'?
a) A gender centric budget which provides equal employment to both men and women.
b) A budget based on environment centric approach and carbon neutral policies.
c) A budget which becomes less viable at times of recession and high unemployment.
d) A budget that allows the government to spend more on public welfare schemes, than on capital formation.

Ans) c
Exp) Option c is correct.
Balanced Budget is a budget where government spend an amount equal to its revenue. This type of budget is
based on the principle of “living within means.” A balanced budget does not ensure financial stability at times of
economic depression or deflation. Balanced budget is unviable at times of recession and does not offer any
solution to problems such as unemployment. It is inapplicable in less developed countries as it limits the scope
of economic growth. It also restricts the government from spending on public welfare.
Source: https://economictimes.indiatimes.com/budget-faqs/what-are-the-three-types-of-government-
budgets/articleshow/67466774.cms?from=mdr

Q.18) A decrease in tax to GDP ratio of a country indicates which of the following?
1. Slowing economic growth rate.
2. Less equitable distribution of national income.
Select the correct answer using the codes given below.
a) 1 only
b) 2 only
c) Both 1 and 2
d) Neither 1 nor 2

Ans) a
Exp) Option a is correct.

Forum Learning Centre: Delhi - 2nd Floor, IAPL House, 19 Pusa Road, Karol Bagh, New Delhi - 110005 | Patna - 2nd floor, AG Palace, E Boring Canal Road,
Patna, Bihar 800001 | Hyderabad - 1st & 2nd Floor, SM Plaza, RTC X Rd, Indira Park Road, Jawahar Nagar, Hyderabad, Telangana 500020
9821711605 | https://academy.forumias.com | admissions@forumias.academy | helpdesk@forumias.academy
Page 10 of 31

SFG 2022 | LEVEL 1 | Test #12 – Solutions |


Statement 1 is correct. Tax to GDP ratio compares the amount of taxes collected by a government to the amount
of income that country receives for its products. A decrease in tax to GDP ratio of a country indicates a slowing
economic growth. When tax revenues grow at a slower rate than the GDP of a country, the tax-to GDP ratio
drops.
Statement 2 is incorrect. A decrease in tax to GDP ratio of a country does not indicate a less equitable distribution
of national income. Tax to GDP ratio and income inequality are not directly corelated with each other.
Source) UPSC 2015

Q.19) In a given economy, taxes are too high which discourages economic activities and thus reduces the tax
revenue for the Government. Government then decided to cut the tax rates that helped in stimulating economic
growth and in increasing tax revenue. If the whole situation is depicted on curve, then the curve will be
a) Laffer Curve
b) Phillips Curve
c) Engel Curve
d) Kuznets Curve

Ans) a
Exp) Option a is correct.
The Laffer Curve is a theory that states lower tax rates boost economic growth and high tax revenue. If taxes are
too high along the Laffer Curve, then they will discourage the taxed activities, such as work and investment,
enough to actually reduce total tax revenue. In this case, cutting tax rates will both stimulate economic incentives
and increase tax revenue.

Option b is incorrect. Phillips curve is an economic concept stats that inflation and unemployment have a stable
and inverse relationship. The theory claims that with economic growth comes inflation, which in turn should lead
to more jobs and less unemployment.
Option c is incorrect. Engel Curve displays how household expenditure on a particular good or service varies
with change in household income. Eg. As income of a household increases its expenditure of food as a percentage
decline. However, its expenditure on status goods increases.
Option d is incorrect. Kuznets Curve shows the relationship between economic growth and inequality. It is
inverted U shaped meaning that as initially economic growth leads to greater inequality, followed later by the
reduction of inequality.
Source: https://www.investopedia.com/terms/l/laffercurve.asp

Q.20) Which one of the following statements is correct about the ‘Consolidated Sinking Fund’?
a) It contains revenues received by the government through taxes and expenses.
b) It is a fund available to State governments for servicing their liabilities.
c) The fund is placed at the disposal of the President to meet unforeseen expenditure.
d) It is a fund that collects money received by the government through small saving instruments.

Forum Learning Centre: Delhi - 2nd Floor, IAPL House, 19 Pusa Road, Karol Bagh, New Delhi - 110005 | Patna - 2nd floor, AG Palace, E Boring Canal Road,
Patna, Bihar 800001 | Hyderabad - 1st & 2nd Floor, SM Plaza, RTC X Rd, Indira Park Road, Jawahar Nagar, Hyderabad, Telangana 500020
9821711605 | https://academy.forumias.com | admissions@forumias.academy | helpdesk@forumias.academy
Page 11 of 31

SFG 2022 | LEVEL 1 | Test #12 – Solutions |


Ans) b
Exp) Option b is correct.
Option a is incorrect: Consolidated Fund of India includes revenues, which are received by the government
through taxes and expenses incurred in the form of borrowings and loans.
Option b is correct: Consolidated Sinking Fund is a fund available to State governments for servicing their
liabilities. Every state government deposits 1% to 3% of its total debt in a consolidated sinking fund to repay the
due loan/due bond payment on maturity date in the future. It is managed by the RBI. At present 23 states have
set up consolidated sinking funds.
Option c is incorrect: Contingency Fund of India is a fund placed at the disposal of the President to meet
unforeseen expenditure pending authorisation of such expenditure by Parliament by law.
Option d is incorrect: Public Account was constituted under Article 266 (2) of the Constitution. It is a fund that
collects money held by the government in a trust. Examples of those are provident funds, small savings.
Expenditures from it are not required to be approved by the Parliament.
Knowledge Base:
The Consolidated Sinking Fund was first proposed by Tenth Finance Commission and was formed in the year
1999-2000 to give a buffer to the State Governments in meeting the future reimbursement commitments.
Twelfth Finance Commission suggested that the credits from banks and liabilities due to the National Small
Saving fund must also be included in the amortization of loans.
The purpose of these measures was to ease the burden that State exchequers may face in nurturing local bodies
to help them attain their potential and discharge their appointed functions.
The fund has to be financed outside the consolidated fund of the states and public account should only be used
for redemption of loans.
Benefits of CSF -
1) Lower default risk.
2) Lower interest rates on the bonds.
3) Help state governments to meet their fiscal deficit.
Source: https://www.legalserviceindia.com/legal/article-2730-explained-consolidated-sinking-fund.html
https://www.financialexpress.com/what-is/consolidated-fund-of-india-meaning/1762621/
https://ccaind.nic.in/govt_accounts.asp

Q.21) With reference to the governance of public sector banking in India, consider the following statements:
1. Capital infusion into public sector banks by the Government of India has steadily increased in the last decade.
2. To put the public sector banks in order, the merger of associate banks with the parent State Bank of India has
been affected.
Which of the statements given above is/are correct?
a) 1 only
b) 2 only
c) Both 1 and 2
d) Neither 1 nor 2

Ans) b
Exp) Option b is correct.
Statement 1 is incorrect. Capital infusion into public sector banks by the Government of India has seen a
fluctuating trend and has not increased steadily.

Forum Learning Centre: Delhi - 2nd Floor, IAPL House, 19 Pusa Road, Karol Bagh, New Delhi - 110005 | Patna - 2nd floor, AG Palace, E Boring Canal Road,
Patna, Bihar 800001 | Hyderabad - 1st & 2nd Floor, SM Plaza, RTC X Rd, Indira Park Road, Jawahar Nagar, Hyderabad, Telangana 500020
9821711605 | https://academy.forumias.com | admissions@forumias.academy | helpdesk@forumias.academy
Page 12 of 31

SFG 2022 | LEVEL 1 | Test #12 – Solutions |

Statement 2 is correct. To put the public sector banks in order, the merger of associate banks with the parent
State Bank of India has been affected. The merger of SBI associated banks under Section 35 of the State Bank of
India Act, 1955 will result in the creation of a stronger merged entity. This will minimize vulnerability to any
geographic concentration risks faced by subsidiary banks. It will create
improved operational efficiency and economies of scale. It will also result in improved risk management and
unified treasury operations.
Source) UPSC 2018

Q.22) With reference to Fiscal Responsibility and Budget Management Act (FRBM), which of the following
statements is correct?
a) According to the FRBM Act, the government can deviate from its annual fiscal deficit target.
b) The government has projected a fiscal deficit of 3 per cent of GDP for financial year 2021-22.
c) The government has projected to eliminate the fiscal deficit by financial year 2025-26.
d) Escape clause under FRBM act cannot be invoked for far-reaching structural reforms in the economy.

Ans) a
Exp) Option a is correct.
Statement a is correct. The FRBM Act allows invoking of an escape clause in situations of calamity and national
security.
In such situations, the government can deviate from its annual fiscal deficit target.
Statement b is incorrect. Redrawing the fiscal consolidation roadmap, the government has projected a fiscal
deficit of 6.8 per cent of Gross Domestic Product (GDP) for financial year 2021-22,
Statement c is incorrect. Redrawing the fiscal consolidation roadmap, the government has projected to reduce
fiscal deficit it to 4.5 per cent by financial year 2025-26.
Statement d is incorrect. Escape clause under FRBM Act can also be invoked for far-reaching structural reforms
in the economy.
Source: https://economictimes.indiatimes.com/markets/stocks/news/what-is-medium-term-fiscal-policy-
statement/articleshow/56737837.cms?utm_source=contentofinterest&utm_medium=text&utm_campaign=cp
pst

Forum Learning Centre: Delhi - 2nd Floor, IAPL House, 19 Pusa Road, Karol Bagh, New Delhi - 110005 | Patna - 2nd floor, AG Palace, E Boring Canal Road,
Patna, Bihar 800001 | Hyderabad - 1st & 2nd Floor, SM Plaza, RTC X Rd, Indira Park Road, Jawahar Nagar, Hyderabad, Telangana 500020
9821711605 | https://academy.forumias.com | admissions@forumias.academy | helpdesk@forumias.academy
Page 13 of 31

SFG 2022 | LEVEL 1 | Test #12 – Solutions |


https://indianexpress.com/article/business/budget/budget-2021-fy22-fiscal-deficit-target-at-6-8-5-year-
glide-path-drawn-to-rein-it-in-7170670/
https://timesofindia.indiatimes.com/business/faqs/budget-faqs/budget-what-is-frbm-act-why-is-it-in-
focus-this-year/articleshow/80454944.cms

Q.23) With reference to Public debt in India, consider the following statements:
1. India's public debt as a percent of GDP has been continuously rising during last decade.
2. Central government's external debt is higher than its internal debt.
3. Central government’s debt to GDP ratio is higher than the combined debt to GDP of states.
Which of the statements given above is/are correct?
a) 1 only
b) 2 and 3 only
c) 3 only
d) 1, 2 and 3

Ans) c
Exp) Option c is correct.
Government debt includes the stock of total liabilities due to internal debt raised through treasury bills, bonds
and securities; external debt mainly raised from multilateral institutions; and public account liabilities such as
provident fund commitments and National Small Savings Fund.
Statement 1 is incorrect. Public debt of India is not steadily rising in the last decade. Outstanding debt is the
accumulation of borrowings over the years. Outstanding debt of the government decreased from 66.7% of GDP
in 2004-05 to 48% of GDP in 2018-19.

Statement 2 is incorrect. Central governments external debt is lower than the internal debt. However, the
external debt to GDP ratio increased to 21.1 per cent at end-March 2021 from 20.6 per cent at end-March 2020.
Over the years, the Union government has followed a considered strategy to reduce its dependence on foreign
loans in its overall loan mix.
Statement 3 is correct. Union government’s debt to GDP is higher than the debt to GDP of states in India. The
Union government’s liabilities account for a little over 46% of the country’s GDP. However, if the public debt is
calculated as general government liabilities, which also includes the liabilities of states then it goes up to 68% of
the country’s GDP. Which means states debt to GDP is 22%.
Source: economic surveys and Government Budget and economy chapter 5: macroeconomics class 12th
https://prsindia.org/files/budget/budget_parliament/2021/Union%20Budget%20Analysis%20-%202021-
22_0.pdf
https://www.rbi.org.in/Scripts/BS_PressReleaseDisplay.aspx?prid=51819
https://www.financialexpress.com/what-is/public-debt-meaning/1627719/

Q.24) Which of the following can be said to be essentially the parts of ‘Inclusive Governance’?
1. Permitting the Non-Banking Financial Companies to do banking
2. Establishing effective District Planning Committees in all the districts
3. Increasing the government spending on public health

Forum Learning Centre: Delhi - 2nd Floor, IAPL House, 19 Pusa Road, Karol Bagh, New Delhi - 110005 | Patna - 2nd floor, AG Palace, E Boring Canal Road,
Patna, Bihar 800001 | Hyderabad - 1st & 2nd Floor, SM Plaza, RTC X Rd, Indira Park Road, Jawahar Nagar, Hyderabad, Telangana 500020
9821711605 | https://academy.forumias.com | admissions@forumias.academy | helpdesk@forumias.academy
Page 14 of 31

SFG 2022 | LEVEL 1 | Test #12 – Solutions |


4. Strengthening the Mid-day Meal Scheme
Select the correct answer using the codes given below:
a) 1 and 2 only
b) 3 and 4 only
c) 2, 3 and 4 only
d) 1, 2, 3 and 4

Ans) c
Exp) Option c is correct.
Statements 2, 3 and 4 are correct. Governance is inclusive when it effectively serves and engages all people; takes
into account gender and other facets of personal identity; and when institutions, policies, processes and services
are accessible, accountable and responsive to all members of society.
Inclusion in terms of both process (how decisions are made and who is included in that process and how and
why) and outcomes (how wealth and prosperity are distributed and shared across a population and why) is a
leading priority in international development, with the Sustainable Development Goals as perhaps the most
ambitious articulation of this. As the evidence overwhelmingly shows, over the long term, more open and
inclusive states and societies tend to be more prosperous, effective and resilient. And yet, it is far less clear how
countries that today can be considered more inclusive in terms of both process and outcome got to where they
are.
Source) UPSC 2012

Q.25) With reference to the borrowing powers of Center and state, consider the following statements:
1. States cannot borrow more than 3.5% of their state GDP under any circumstance.
2. A state cannot borrow within India without the Centre’s consent if it has any loan outstanding to the
Government of India.
Which of the statements given above is/are correct?
a) 1 only
b) 2 only
c) Both 1 and 2
d) Neither 1 nor 2

Ans) b
Exp) Option b is correct.
Statement 1 is incorrect. Earlier, states were not allowed to borrow more than 3.5% of their GDP. Recently, the
Centre has raised borrowing limits for states to 5% of gross state domestic product (GSDP). The raised limit is
subject to carrying out of specific reform by the states.
Statement 2 is correct. Under Article 293 (3) of the Indian Constitution a state can’t raise borrowings within
country without the Centre’s consent if it has any loan outstanding to the Government of India.
Source: https://www.newindianexpress.com/business/2020/aug/29/hands-tied-states-unable-to-tick-
borrowing-option-2189686.html

Q.26) With reference to Ricardian Equivalence Proposition, which of the following statement is correct?
a) It states that government spending through borrowing is equivalent to its spending out of tax revenue.
b) It proposes that government should boost the economy by deficit financing.
c) It means that government should borrow money from the central bank to prevent crowding out of private
investments.
d) It states that central bank should avoid funding the deficit of the Government.

Ans) a
Exp) Option a is correct.

Forum Learning Centre: Delhi - 2nd Floor, IAPL House, 19 Pusa Road, Karol Bagh, New Delhi - 110005 | Patna - 2nd floor, AG Palace, E Boring Canal Road,
Patna, Bihar 800001 | Hyderabad - 1st & 2nd Floor, SM Plaza, RTC X Rd, Indira Park Road, Jawahar Nagar, Hyderabad, Telangana 500020
9821711605 | https://academy.forumias.com | admissions@forumias.academy | helpdesk@forumias.academy
Page 15 of 31

SFG 2022 | LEVEL 1 | Test #12 – Solutions |


Option a is correct. Ricardian equivalence is an economic theory that says that financing government spending
out of current taxes or future taxes (and current deficits) will have equivalent effects on the overall economy.
Ricardian equivalence maintains that government deficit spending is equivalent to spending out of current taxes.
This means that attempts to stimulate an economy by increasing debt-financed government spending will not
be effective because investors and consumers understand that the debt will eventually have to be paid for in the
form of future taxes. Because taxpayers will save to pay the expected future taxes, this will tend to offset the
macroeconomic effects of increased government spending.
Source:
https://www.investopedia.com/terms/r/ricardianequivalence.asp
Government Budget and economy chapter 5: macroeconomics class 12th

Q.27) Consider the following statements:


1. Tax revenue as a percent of GDP of India has steadily increased in the last decade.
2. Fiscal deficit as a percent of GDP of India has steadily increased in the last decade.
Which of the statements given above is/are correct?
a) 1 only
b) 2 only
c) Both 1 and 2
d) Neither 1 nor 2

Ans) d
Exp) Option d is correct.
Both Statements 1 and 2 are incorrect.

The ratio of taxes -to-GDP slid further in FY20 to a 10-year low of 9.88 per cent, driven by a decline in collections
from customs duties and corporation tax, while excise duty posted marginal growth. India's gross tax-to-GDP
ratio fell to 10.9 per cent in 2018-19 on account of lower than estimated GST collection. The ratio stood at 11.22
per cent in FY18.
Fiscal deficit is the gap between total expenditure and total income of the government.

Forum Learning Centre: Delhi - 2nd Floor, IAPL House, 19 Pusa Road, Karol Bagh, New Delhi - 110005 | Patna - 2nd floor, AG Palace, E Boring Canal Road,
Patna, Bihar 800001 | Hyderabad - 1st & 2nd Floor, SM Plaza, RTC X Rd, Indira Park Road, Jawahar Nagar, Hyderabad, Telangana 500020
9821711605 | https://academy.forumias.com | admissions@forumias.academy | helpdesk@forumias.academy
Page 16 of 31

SFG 2022 | LEVEL 1 | Test #12 – Solutions |

Kb) Tax-to-GDP ratio represents the size of a country's tax kitty relative to its GDP. It is a representation of the
size of the government's tax revenue expressed as a percentage of the GDP. A higher tax to GDP ratio means that
the government is able to cast its fiscal net wide. It reduces a government's dependence on borrowings.
Source) UPSC Prelims 2017

Q.28) Which of the following is the most likely impact of Direct Monetization of government deficit on the
economy?
a) Shifting the burden of high taxation from future generations to current generation.
b) Increase in unemployment
c) Reduction in aggregate demand in the economy
d) Increase in the level of inflation

Ans) d
Exp) Option d is correct.
Direct Monetization refers to the scenario where the central bank accommodates the government’s fiscal deficit
by printing new currency equivalent to that amount. The central bank does this by buying government issued
securities directly from the primary market.
Statement a is incorrect: Direct Monetization is like a debt taken by the present day government, which has to
be paid off ultimately at some point of time in the long run. So whenever this debt is to be paid off the then
government of the future can do so only by increasing the resources they collect from the citizens in the form of
taxes. Thus, shifting the burden of high taxation from present generations to future generation.
Statement b is incorrect: Due to monetization of the deficit, the government can spend as much as it wants as
its deficits are easily balanced by printing of currency by the central bank. So the government is free to follow an
expansionary fiscal policy. This means the government spends more on direct employment generation programs
like MGNREGA, it can give out more contracts under PPP mode, etc thus increasing investment in the economy,
thus creating more job opportunities (not unemployment), as businesses expand due to increasing investment.
Statement c is incorrect: Deficit Monetization increases (not decreases) the aggregate demand in an economy.
Since there is increased cash flowing through the economy, due to printing of currency equivalent to the amount
of government deficit, it reaches various economic agents like businesses and consumers who use it to buy
various economic goods, thus increasing the demand.
For example, as the government will have a huge deficit, it will use this money to create infra like roads via PPP
method, so it will hire a private enterprise for the job, who will need to buy more materials for road construction,
thus raising the demand of materials involved in road construction.
Statement d is correct: The Direct Monetization of deficit occurs by printing more of the currency by the central
bank. It thus increases the cash in circulation in an economy, which causes increased demand, which outstrips
supply, thus leading to an increase in prices, i.e. increasing level of inflation.
Knowledge Base:

Forum Learning Centre: Delhi - 2nd Floor, IAPL House, 19 Pusa Road, Karol Bagh, New Delhi - 110005 | Patna - 2nd floor, AG Palace, E Boring Canal Road,
Patna, Bihar 800001 | Hyderabad - 1st & 2nd Floor, SM Plaza, RTC X Rd, Indira Park Road, Jawahar Nagar, Hyderabad, Telangana 500020
9821711605 | https://academy.forumias.com | admissions@forumias.academy | helpdesk@forumias.academy
Page 17 of 31

SFG 2022 | LEVEL 1 | Test #12 – Solutions |


1) This method of Direct monetization was the default uptil 1997. Thereafter it was phased out, and completely
ended in 2003, with the introduction of Fiscal Responsibility and Budget Management Act, 2003, with a n aim
of improving fiscal discipline and improving government’s quality of expenditures.
2) in 2017, an escape clause was added to this Act to allow government to access this route of budget
management and resource mobilisation in case of emergencies like natural disasters, war, etc
Source: https://www.livemint.com/opinion/online-views/opinion-why-or-why-not-direct-monetisation-of-
deficits-should-wait-in-india-11597730356098.html
https://economictimes.indiatimes.com/markets/stocks/news/deficit-monetisation-is-it-really-as-simple-
as-rbi-printing-more-money/articleshow/75968757.cms?from=mdr

Q.29) With reference to Off Budget Borrowings, consider the following statements:
1. It refers to the borrowings made by the Central Public Sector Enterprises on behalf of the Union Government.
2. They are counted as a part of Fiscal Deficit.
3. The interest accrued on these are paid entirely from the government budget.
4. It reduces Parliamentary Control over the Executive in financial matters.
Which of the statements given above is/ are correct?
a) 1, 2 and 4 only
b) 3 and 4 only
c) 1, 3 and 4 only
d) 1, 2, 3 and 4

Ans) c
Exp) Option c is correct.
Statement 1 is correct: Off Budget Borrowings are the loans taken by government owned entities like Central
Public Sector Enterprises (CPSEs) on the directions of the government in order to supplement the expenditure
made by the government.
For example, in the Budget presentation for 2020-21, the government paid only half the amount budgeted for the
food subsidy bill to the Food Corporation of India. The shortfall was met through a loan from the National Small
Savings Fund. This allowed the Centre to halve its food subsidy bill from Rs 1,51,000 crore to Rs 77,892 crore in
2020-21; loans from PSU banks were used to make up for the shortfall in the release of fertiliser subsidy, etc
Statement 2 is incorrect: Even though these borrowings have implications on government finances and
macroeconomic stability, they are not included in the Fiscal Deficit. This is the reason they are called Off Budget
or Extra-Budgetary Borrowings.
Statement 3 is correct: The interest as well as the principal amount incurred through such borrowings is the
liability of the Central Government and has to be paid from government resources/ budget itself (Consolidated
Fund of India). This is why off budget borrowings have implications for public finances and macroeconomic
stability.
Statement 4 is correct: The Parliament controls the Executive, that is the Government by restricting their
appropriation of public funds collected via taxes, fees, etc from the citizens, from the Consolidated Fund of India.
However if the Executive uses alternative routes like directing CPSEs to take loans, which are not a part of deficits
or budget calculations. The legislature will not thus be able to question or control government expenditure or its
quality (productive or wasteful), thus reducing their accountability to citizens.
This will also create a danger to macroeconomic stability as the FRBM Act won’t apply to these borrowings thus
causing disturbance in fiscal discipline and making the government careless with expenditure at the cost of
higher interests with passing time.
Knowledge Base:
1) These are excluded from the Fiscal Deficit of the government but are indeed a part of the Total Debt of the
government.
2) These are raised from the market by PSEs by using Government of India Fully Serviced Bonds
Source: https://www.indianeconomy.net/splclassroom/what-are-extra-budgetary-off-budget-borrowings/
https://indianexpress.com/article/explained/why-govt-borrows-off-budget-and-how-7162925/

Forum Learning Centre: Delhi - 2nd Floor, IAPL House, 19 Pusa Road, Karol Bagh, New Delhi - 110005 | Patna - 2nd floor, AG Palace, E Boring Canal Road,
Patna, Bihar 800001 | Hyderabad - 1st & 2nd Floor, SM Plaza, RTC X Rd, Indira Park Road, Jawahar Nagar, Hyderabad, Telangana 500020
9821711605 | https://academy.forumias.com | admissions@forumias.academy | helpdesk@forumias.academy
Page 18 of 31

SFG 2022 | LEVEL 1 | Test #12 – Solutions |


Q.30) If a commodity is provided free to the public by the Government, then
a) the opportunity cost is zero.
b) the opportunity cost is ignored.
c) the opportunity cost is transferred from the consumers of the product to the tax-paying public.
d) the opportunity cost is transferred from the consumers of the product to the Government.

Ans) c
Exp) Option c is correct.
Opportunity cost is the forgone benefit that would have been derived by an option not chosen. For example, if
you decide to spend two hours studying on a Friday night. The opportunity cost is that you cannot have those
two hours for leisure.
When a commodity is provided free to the public by the government then the opportunity cost is transferred to
the tax-paying public.
Source) UPSC 2018

Q.31) With reference to Public Goods, consider the following statements:


1. They are non-excludable for consumers.
2. Consumption of public goods by one consumer necessarily reduces the availability for other consumers
3. Electricity distributed by government to houses is an example of public good.
Which of the statements given above is/ are correct?
a) 1 only
b) 2 and 3 only
c) 1 and 3 only
d) 1, 2 and 3

Ans) a
Exp) Option a is correct.
Statement 1 is correct: Public Goods are non-Excludable. This means that even if some consumers don’t pay for
the creation of these goods, they can’t be stopped from utilising or excluded from deriving the benefits of these
goods.
Statement 2 is incorrect: For Private Goods, the consumption by one consumer reduces the availability of said
goods for other consumers. But the claims of consumers over public goods create no rivalries in availability for
consumption as the benefits of such goods cannot be limited to few.
For example, the benefit of trees planted in cities to reduce temperature and air pollution cannot be restricted
to tax paying citizens only.
Statement 3 is incorrect: Electricity is not a public good. Because electricity is provided to the people who pay
for it. However, roads, parks, street lights, etc are examples of public goods.
Knowledge Base: These goods cannot be provided by the private sector which operates on a profit motive. They
are provided by the government as a part of its Allocative role in the economy.
Source: Government Budget and economy chapter 5: macro economics class 12th

Q.32) Which of the following is a feature of the National Monetization Pipeline?


a) It includes monetisation of only greenfield projects.
b) The monetisation of the assets will be conducted through Infrastructure Investment Trusts.
c) Oil and Gas pipelines form the biggest sector by value in the Pipeline
d) There will be a transfer of ownership rights to the private sector partner.

Ans) b
Exp) Option b is correct.
The National Monetisation Pipeline (NMP) is envisaged to serve as a medium-term roadmap for identifying
potential monetisation- ready projects, across various infrastructure sectors. It aims to unlock the value of

Forum Learning Centre: Delhi - 2nd Floor, IAPL House, 19 Pusa Road, Karol Bagh, New Delhi - 110005 | Patna - 2nd floor, AG Palace, E Boring Canal Road,
Patna, Bihar 800001 | Hyderabad - 1st & 2nd Floor, SM Plaza, RTC X Rd, Indira Park Road, Jawahar Nagar, Hyderabad, Telangana 500020
9821711605 | https://academy.forumias.com | admissions@forumias.academy | helpdesk@forumias.academy
Page 19 of 31

SFG 2022 | LEVEL 1 | Test #12 – Solutions |


investment made in public assets which have not yielded appropriate or potential returns so far and create
hitherto unexplored sources of income for the company and its shareholders.
Statement a is incorrect. National Monetisation Pipeline aims to unlock value in brownfield projects by engaging
the private sector. Then, using the funds generated for infrastructure creation across the country.
National Monetisation Pipeline involves pre-existing and stable brownfield (not greenfield) projects. This has
been done to encourage the private sector to participate as these prime and well-functioning government assets
are de-risked and offer more chances of success and profit creation to the private sector.
Statement b is correct: The monetisation of the assets under the NMP will be done through direct contractual
instruments (such as public private partnership concessions) and capital market instruments (such as
Infrastructure Investment Trusts). The monetisation value that is expected to be realised may either be in form
of upfront accruals or by way of private sector investment.
Statement c is incorrect: Roads (not Oil & Gas Pipelines) at 27% constitute the biggest share in the sectors whose
assets have been put forward for monetization. It is closely followed by Railways (25%), Power (15%), oil & gas
pipelines (8%) and Telecom (6%). These top 5 sectors account for 83% of pipeline value.
Statement d is incorrect: This monetization scheme has been designed where the revenue collection and
appropriation rights from the government owned assets will be transferred to the private sector partner. There
will be no transfer of ownership rights, and in the end, the public sector assets will be handed over back to the
government.
Knowledge Base: NMP aims to unlock the value in brownfield projects by partnering with the private sector, to
transfer to them only the revenue rights and not ownership in the projects, and using the funds generated for
infrastructure creation across the country.
Its total value is estimated to be around 6 lakh Crore.
This current FY 2021-22, 15% of assets with an indicative value of Rs 0.88 lakh crore are expected to be monetized.
Kindly Read Further:
https://indianexpress.com/article/explained/explained-what-is-the-governments-plan-with-the-national-
monetisation-pipleline-7468258/
Source: https://www.india.gov.in/spotlight/national-monetisation-pipeline-nmp
https://indianexpress.com/article/explained/explained-what-is-the-governments-plan-with-the-national-
monetisation-pipleline-7468258/

Q.33) Consider the following statements:


1. Coal sector was nationalized by the Government of India under Indira Gandhi.
2. Now, coal blocks are allocated on lottery basis.
3. Till recently, India imported coal to meet the shortages of domestic supply, but now India is self-sufficient in
coal production.
Which of the statements given above is/are correct?
a) 1 only
b) 2 and 3 only
c) 3 only
d) 1, 2 and 3

Ans) a
Exp) Option a is correct.
Statement 1 is correct. Indira Gandhi government nationalized coal sector in 1971-73 period. This was reversed
by NDA government in 2015, opening the sector to private players.
Statement 2 is incorrect. Under the new policy, the allocation is done on the basis of bidding process-the firm
offering the highest per tonne price is allotted the mine.
Statement 3 is incorrect. Though India has fifth largest reserve coal reserve in the world, it is not self-sufficient
as it has to import high quality coal to meet its domestic demand.
Source) UPSC 2019

Forum Learning Centre: Delhi - 2nd Floor, IAPL House, 19 Pusa Road, Karol Bagh, New Delhi - 110005 | Patna - 2nd floor, AG Palace, E Boring Canal Road,
Patna, Bihar 800001 | Hyderabad - 1st & 2nd Floor, SM Plaza, RTC X Rd, Indira Park Road, Jawahar Nagar, Hyderabad, Telangana 500020
9821711605 | https://academy.forumias.com | admissions@forumias.academy | helpdesk@forumias.academy
Page 20 of 31

SFG 2022 | LEVEL 1 | Test #12 – Solutions |


Q.34) Which of the following types of taxes are being levied in India currently?
1. Wealth Tax
2. Dividend Distribution Tax
3. Long Term Capital Gain Tax
4. Gift Tax
5. Fringe Benefit Tax
Select the correct option using the code given below:
a) 1, 2 and 3 only
b) 3 and 4 only
c) 1, 2, 3 and 5 only
d) 1, 3, 4 and 5 only

Ans) b
Exp) Option b is correct.
Option 1 is incorrect: Wealth tax was levied as per the Wealth tax Act, 1957. It taxed the extremely rich taxable
entities like individuals, Companies and Hindu Undivided Families at 1% of their wealth above 30 lakhs of rupees.
Taxable wealth under this tax included resident Indian’s global assets and NRIs’ Indian assets. The intention was
to bring parity among various citizens of the country.
It was abolished in the Finance Act of 2015 as the cost for recovering this tax exceeded the amount collected by
it.
Option 2 is incorrect: Dividend Distribution Tax was the tax levied on a Company whenever it declares,
distributes or pays any amount as a dividend, at a rate of 15%. The Finance Act, 1997 introduced the provisions of
DDT. However it has been abolished since 2020. The dividend distribution tax (DDT) has been abolished at both
the company and mutual fund level.
Option 3 is correct: Long Term Capital Gains Tax is a tax imposed at a rate of 20% on any profits that are made
by individuals upon selling capital assets (art, property, etc) after a period of 3 years from its purchase (1 year in
case of equity).
Option 4 is correct: Gift Tax is regulated by the Gift Tax Act 1958. It taxes any movable or immovable property
given from one person to another, including relatives, without the return of any goods and services (act of gifting).
It exempts gifts given on special occasions like marriages, succession, donation, etc. This has been done so that
gifting may not be used to evade taxes.
Option 5 is incorrect: Fringe Benefit Tax is a tax levied on the perks and benefits provided by Companies to their
employees. It was paid by employers to the government. It was introduced in 2005 and was abolished by the
budget in FY 2010-11. When in force it was applied at a flat rate of 30%.
It has been replaced in a slightly changed form by the Perquisites Tax.
Knowledge Base:
Some other taxes levied in India -
1) Securities Transaction Tax
2) Income tax
3) Minimum Alternative Tax
Source: https://cleartax.in/s/wealth-tax
https://www.incometaxindia.gov.in/Tutorials/41.%20Wealth_Tax.pdf
https://cleartax.in/s/how-are-gifts-taxed
https://www.business-standard.com/about/what-is-fringe-benefit-tax-fbt
https://www.bankbazaar.com/tax/fringe-benefit-tax.html
https://cleartax.in/s/how-dividends-taxable
https://www.incometaxindia.gov.in/Pages/i-am/domestic-
company.aspx?k=Dividend%20Distribution%20Tax
https://groww.in/p/long-term-capital-gains-tax/
https://www.bankbazaar.com/tax.html#indirect-tax

Forum Learning Centre: Delhi - 2nd Floor, IAPL House, 19 Pusa Road, Karol Bagh, New Delhi - 110005 | Patna - 2nd floor, AG Palace, E Boring Canal Road,
Patna, Bihar 800001 | Hyderabad - 1st & 2nd Floor, SM Plaza, RTC X Rd, Indira Park Road, Jawahar Nagar, Hyderabad, Telangana 500020
9821711605 | https://academy.forumias.com | admissions@forumias.academy | helpdesk@forumias.academy
Page 21 of 31

SFG 2022 | LEVEL 1 | Test #12 – Solutions |


Q.35) Consider the following statements regarding various public expenditures:
1. A public expenditure is progressive if it benefits only the lower income bracket people.
2. Providing subsidy on private savings is an example of regressive expenditure.
3. Proportional expenditure helps to reduce inequalities in distribution of income.
Which of the statements given above is/are correct?
a) 1 and 2 only
b) 2 only
c) 2 and 3 only
d) 1 and 3 only

Ans) b
Exp) Option b is correct.
Expenses incurred by the public authorities—central, state and local self- governments—are called public
expenditure. Such expenditures are made for the maintenance of the governments as well as for the benefit of
the society as whole.
Statement 1 is incorrect. A public expenditure is progressive, if the additional benefit provided by the grant or
public expenditure is larger in the case of low income people and lower in the case of high income people. It does
not mean that it only benefits the lower income people. For example- expenditure on social security like free
medical aid, free education, subsidized houses etc. are cases of progressive expenditure. It reduces income
inequality existing in the society.
Statement 2 is correct. A public expenditure is regressive, if it benefits more to the higher income beneficiary
than smaller income beneficiary. For example, subsidy on private saving is regressive in nature because private
savings will be more for high income group compared to low-income group. Such subsidy will thus benefit more
to the higher income group. It increases inequality.
Statement 3 is incorrect. The public expenditure is said to be proportional, when the proportion of additional
benefit provided by the grant or public expenditure is the same, whatever the size of the recipient income. Here,
there is no change in existing inequalities of income distribution. For example, if all categories of employees were
given a house allowance at 10 % of their salaries, then it is a case of proportional expenditure.
Source: https://www.msuniv.ac.in/Download/Pdf/4f8c172a46694f7
http://www.lscollege.ac.in/sites/default/files/e-
content/15.%20public%20Expenditure%20and%20Distribution.pdf
https://discover.hubpages.com/education/Effects-of-Public-Expenditure

Q.36) In the context of independent India’s economy, which one of the following was the earliest event to take
place?
a) Nationalization of Insurance companies
b) Nationalization of State Bank of India
c) Enactment of Banking Regulation Act
d) Introduction of First Five-Year Plan

Ans) c
Exp) Option c is correct.
Nationalization of State Bank of India – 1955;
Introduction of First Five-Year Plan – 1951;
Enactment of Banking Regulation Act – 1949;
Nationalization of Insurance Companies – 1955-56
Source) UPSC Prelims 2009

Q.37) Which of the following statements is correct regarding the Post Devolution Revenue Deficit Grant?
a) It is a way of providing grants to states for development related projects and schemes.
b) These are provided to all states to meet their revenue deficits.

Forum Learning Centre: Delhi - 2nd Floor, IAPL House, 19 Pusa Road, Karol Bagh, New Delhi - 110005 | Patna - 2nd floor, AG Palace, E Boring Canal Road,
Patna, Bihar 800001 | Hyderabad - 1st & 2nd Floor, SM Plaza, RTC X Rd, Indira Park Road, Jawahar Nagar, Hyderabad, Telangana 500020
9821711605 | https://academy.forumias.com | admissions@forumias.academy | helpdesk@forumias.academy
Page 22 of 31

SFG 2022 | LEVEL 1 | Test #12 – Solutions |


c) It is provided to those states which incurred losses on account of better performance in population control
measures.
d) These are released in monthly instalments as per the recommendations of the Finance Commission.

Ans) d
Exp) Option d is correct.
The Post Devolution Revenue Deficit Grants are provided to the States under Article 275 of the Constitution.
Statement a is incorrect. Post Devolution Revenue Deficit Grants are not provided for development related
projects and schemes. These are provided to states to meet the gap in Revenue Accounts of the states post
devolution.
Statement b is incorrect. The Finance Commission decides the state’s eligibility to receive this grants. This is
based on the gap between assessment of revenue and expenditure of the State after taking into account the
assessed devolution for the current financial year. So, it is not provided to all states.
Statement c is incorrect. The eligibility of States to receive this grant and the quantum of grant was decided by
the Commission based on the gap between assessment of revenue and expenditure of the State after taking into
account the assessed devolution.
Statement d is correct. The grants are released in monthly instalments as per the recommendations of the
Finance Commission to meet the gap in Revenue Accounts of the States post devolution
Knowledge Base: The Post Devolution Revenue Deficit Grants are provided to the States under Article 275 of the
Constitution
Source:
https://pib.gov.in/PressReleseDetailm.aspx?PRID=1695495#:~:text=The%20Post%20Devolution%20Revenue%
20Deficit%20Grants%20are%20provided,Commission%20has%20recommended%20PDRD%20grants%20to%2
014%20States.
https://www.financialexpress.com/what-is/finance-commission-grants-and-other-transfers-
meaning/1627710/

Q.38) With regard to the surplus income of RBI, which of the following statements is/are incorrect?
1. RBI is exempted from paying income tax on its surplus income, but has to pay wealth tax.
2. The amount of transfer of surplus to the Centre is decided solely by the government.
3. Surplus transfer to the Government forms part of its capital receipts.
4. Banking Regulations Act of 1949 provides for the transfer of surplus income by the RBI to the Centre.
Select the correct answer using the code given below:
a) 1, 2 and 4 only
b) 2 and 3 only
c) 3 only
d) 1, 2, 3 and 4

Ans) d
Exp) Option d is correct.
The Reserve Bank of India (RBI) transfers its surplus profits to the Government every year. Surplus profit is the
profit left after making various contingency provisions for bad and doubtful debts, depreciation in assets,
contributions to staff, and superannuation funds, etc.
Statement 1 is incorrect. RBI is exempted from paying income tax or any other tax, including wealth tax. Section
48 (Exemption of Bank from income-tax and super-tax) of the RBI Act, 1934 provides that the Bank shall not be
liable to pay income-tax or super-tax on any of its income, profits or gains.
Statement 2 is incorrect. The amount of transfer of surplus to the Centre is decided by the central bank after
consultations with the government. The decision to transfer the surplus is taken at the meeting of the Central
Board of Directors of RBI.
Statement 3 is incorrect. Surplus transfer is put under the head ‘non-tax revenue’ in the revenue budget as it is
dividend. It helps the government narrow its deficit or borrowings.

Forum Learning Centre: Delhi - 2nd Floor, IAPL House, 19 Pusa Road, Karol Bagh, New Delhi - 110005 | Patna - 2nd floor, AG Palace, E Boring Canal Road,
Patna, Bihar 800001 | Hyderabad - 1st & 2nd Floor, SM Plaza, RTC X Rd, Indira Park Road, Jawahar Nagar, Hyderabad, Telangana 500020
9821711605 | https://academy.forumias.com | admissions@forumias.academy | helpdesk@forumias.academy
Page 23 of 31

SFG 2022 | LEVEL 1 | Test #12 – Solutions |


Statement 4 is incorrect. It is not the Banking regulations act 1949, but the section 47 of RBI Act 1935 which
provides for the surplus transfer income of RBI to the Centre.
Knowledge Base:
Source of RBI’s income –
1) Returns it earns on its foreign currency assets which could be in the form of bonds and treasury bills of other
central banks or top-rated securities, and deposits with other central banks.
2) interest on its holdings of local rupee-denominated government bonds or securities, and while lending to
banks for very short tenures, such as overnight.
3) management commission on handling the borrowings of state governments and the central government.
Expenditure of RBI-
1) expenses on the printing of currency notes
2) expenses on staff
3) the commission it gives to banks for undertaking transactions on behalf of the government across the country,
and to primary dealers, including banks, for underwriting some of these borrowings.
Source: https://indianexpress.com/article/explained/explained-why-and-how-does-the-rbi-transfer-
surplus-to-government-5942031/
https://www.business-standard.com/article/economy-policy/reserve-bank-of-india-transfer-tops-govt-s-
non-tax-revenue-source-
121060800036_1.html#:~:text=Dividend%2C%20or%20surplus%20transfer%2C%20from,of%20nearly%20Rs%2
078%2C000%20crore.

Q.39) Which one of the following is responsible for the preparation and presentation of Union Budget to the
Parliament?
a) Department of Revenue
b) Department of Economic Affairs
c) Department of Financial Services
d) Department of Expenditure

Ans) b
Exp) Option b is correct.
Under the Finance Ministry, the Budget Division of the Department of Economic Affairs is the nodal body which
is directly responsible for the formulation of the Union Budget.
Source) UPSC Prelims 2009

Q.40) Occurrence of which of the following events leads to Fiscal drag in the economy?
1. When higher tax rate increases the government’s tax revenue.
2. Individuals moved into higher tax bracket due to inflation.
3. Income growth pushes an individual to higher tax bracket.
4. Consumer spending increases due to high inflation.
Select the correct answer using the code given below:
a) 1, 2 and 3 only
b) 2, 3 and 4 only
c) 2 and 3 only
d) 1 and 4 only

Ans) c
Exp) Option c is correct.
Fiscal drag is an economic term whereby inflation or income growth moves taxpayers into higher tax brackets.
This in effect increases government tax revenue without actually increasing tax rates. The increase in taxes
reduces aggregate demand and consumer spending from taxpayers as a larger share of their income now goes to
taxes, which leads to deflationary policies, or drag, on the economy.

Forum Learning Centre: Delhi - 2nd Floor, IAPL House, 19 Pusa Road, Karol Bagh, New Delhi - 110005 | Patna - 2nd floor, AG Palace, E Boring Canal Road,
Patna, Bihar 800001 | Hyderabad - 1st & 2nd Floor, SM Plaza, RTC X Rd, Indira Park Road, Jawahar Nagar, Hyderabad, Telangana 500020
9821711605 | https://academy.forumias.com | admissions@forumias.academy | helpdesk@forumias.academy
Page 24 of 31

SFG 2022 | LEVEL 1 | Test #12 – Solutions |


Statement 1 is incorrect and statement 2 and 3 are correct. Fiscal drag occurs when income growth and inflation
push more earners into higher tax brackets. Thus, the government’s tax revenue rises without any increases in
tax rates.
Statement 4 is incorrect. Due to progressive taxation, a greater number of people comes in high bracket of
taxation. Thus, Consumer spending decreases besides rise in income growth.
Knowledge Base:
It can be seen as an automatic fiscal stabilizer as it controls a rapidly expanding economy from overheating
Source: https://www.investopedia.com/terms/f/fiscal-drag.asp
https://marketbusinessnews.com/financial-glossary/fiscal-drag/

Q.41) Which of the following means are available for deficit financing in India?
1. External aid in the form of grants
2. Borrowing money from domestic market.
3. Printing of new money.
4. Ways and Means Advances
5. Borrowing money from foreign markets.
Select the correct code from the options given below:
a) 1,2,3 and 4 only
b) 1,3 and 4 only
c) 1,2,3 and 5 only
d) 1,2,3,4 and 5

Ans) d
Exp) Option d is correct.
Deficit financing means generating funds to finance the deficit which results from excess of expenditure over
revenue.
Statement 1 is correct. External aids and grants are considered a good means of fulfilling government’s deficit
requirements as it does not involve any interest costs.
Statement 3 is correct. Printing Currency is the last resort for the government in managing its deficit. But with
it the government cannot go for the expenditures which are to be made in the foreign currency. Also printing
fresh currencies does have other damaging effects on the economy like inflation.
Statement 2 and 5 is correct. The Government ordinarily prefers to borrow either from its citizens or from foreign
governments instead of withdrawing cash balances held with the R.B.I. or borrowing from it. Borrowing
domestically from public has no effect on the supply of money and consequently on prices because when
government borrows, the money held by people is transferred to government with no change in the supply of
money. However, the money supply would increase when government borrows from foreign countries.
Statement 4 is correct. Ways and Means Advances (WMA) Scheme was introduced in 1997. These are temporary
loan facilities provided by the Reserve Bank of India (RBI) to the central and state governments to meet
mismatches in the receipts and payments.
Knowledge Base:
Limits for WMA are decided by the government and RBI mutually and revised periodically.
Types of WMA: a) Special WMA: It is extended against the collateral of the government securities held by the
State Government and b) Normal WMA: It is based on a three-year average of actual revenue and capital
expenditure of the state. It is not collateral based.
Source: NCERT Class 12th Introductory Macroeconomics – Chapter 5 - Government Budget and economy, Page
– 80.
https://blog.forumias.com/ways-and-means-advances/
https://nios.ac.in/media/documents/SrSec318NEW/318_Economics_Eng/318_Economics_Eng_Lesson29.p
df
https://economictimes.indiatimes.com/budget-faqs/what-is-deficit-financing/articleshow/73329358.cms

Forum Learning Centre: Delhi - 2nd Floor, IAPL House, 19 Pusa Road, Karol Bagh, New Delhi - 110005 | Patna - 2nd floor, AG Palace, E Boring Canal Road,
Patna, Bihar 800001 | Hyderabad - 1st & 2nd Floor, SM Plaza, RTC X Rd, Indira Park Road, Jawahar Nagar, Hyderabad, Telangana 500020
9821711605 | https://academy.forumias.com | admissions@forumias.academy | helpdesk@forumias.academy
Page 25 of 31

SFG 2022 | LEVEL 1 | Test #12 – Solutions |


Q.42) What is the difference between “vote-on-account” and “interim budget”?
1. The provision of a “vote-on-account’’ is used by a regular Government, while an “interim budget’’ is a provision
used by a caretaker Government
2. A “vote-on-account’’ only deals with the expenditure in Government budget, while an “interim budget’’ includes
both expenditure and receipts.
Select the correct answer using the code given below:
a) 1 only
b) 2 only
c) Both 1 and 2
d) Neither 1 nor 2

Ans) b
Exp) Option b is correct.
Statement 1 is incorrect. Vote on account and interim budget can be presented by both the regular government
and the caretaker government.
An interim budget is presented by the government when it does not have time to present a full budget. Mostly, it
is because the general election is around the corner.
Vote-on-Account is a special provision by which the government obtains Parliament’s approval for funds
sufficient to incur expenditure for a part of the year (till the formation of a new government) enabling it to incur
expenses till a full budget is prepared.
Vote-On-Account represents the expenditure side of the government's budget while general budget includes
both income and expenditure in the form of Financial and Appropriation Bill.
Statement 2 is correct. A “vote-on-account’’ only deals with the expenditure in Government budget, while an
“interim budget’’ includes both expenditure and receipts. Vote on account is just an interim permission to spend
money as against an interim Budget which is an elaborate financial statement of expenditure and receipts
including changes in taxes and government policies.
Source) UPSC Prelims 2011

Q.43) With reference to the Gender responsive budgeting in India, consider the following statements:
1. The Union Government introduced the Gender Budget Statement in the Budget of 2005-06.
2. It creates separate funds for programmes dedicated to females.
3. The Gender Budget as a percentage of total budget has not much increased since its introduction.
4. All the states in India have adopted gender budgeting.
Which of the statements given above are correct?
a) 1 and 2 only
b) 2 and 3 only
c) 1 and 3 only
d) 3 and 4 only

Ans) c
Exp) Option c is correct.
Gender budgeting means incorporating a gender perspective at all levels of the budgetary process and
restructuring revenues and expenditures in order to promote gender equality.
Statement 1 is correct. The Government of India (GoI) introduced the Gender Budget Statement (GBS) in the
Union Budget in 2005-06. It constitutes two parts- Part A reflects women-specific schemes having 100%
allocation for women, and Part B reflects pro-women schemes, where at least 30% of the allocation is for women.
Statement 2 is incorrect. Gender responsive budgeting does not entail separate allocation of funds towards
programmes dedicated to women’s affairs. It is rather an approach to budgeting that recognizes the
differentiated impact of budget on men and women. It utilises policy guidelines and tools to prepare budgets that
are more sensitive towards the needs of women, with a focus on alleviating gender inequality.
Statement 3 is correct. In 2005-06, the Gender Budget constituted 4.8% of the total budget outlay. Over the
years this has remained stagnated, hovering around 5%. In the last financial year, the Gender Budget stood at Rs

Forum Learning Centre: Delhi - 2nd Floor, IAPL House, 19 Pusa Road, Karol Bagh, New Delhi - 110005 | Patna - 2nd floor, AG Palace, E Boring Canal Road,
Patna, Bihar 800001 | Hyderabad - 1st & 2nd Floor, SM Plaza, RTC X Rd, Indira Park Road, Jawahar Nagar, Hyderabad, Telangana 500020
9821711605 | https://academy.forumias.com | admissions@forumias.academy | helpdesk@forumias.academy
Page 26 of 31

SFG 2022 | LEVEL 1 | Test #12 – Solutions |


1,43,462 crore or 4.72% of the total budget. The post-pandemic situation continues to be disproportionately
disadvantageous for women, and the Gender Budget as a proportion has fallen to 4.4% in the 2021-22 Union
Budget, which is Rs 1,53,326 in absolute terms.
Statement 4 is incorrect. Even though the government adopted gender budgeting in 2005-06, nine states and
Union Territories — Goa, Telangana, Haryana, Chandigarh, Meghalaya, Ladakh, Mizoram, Puducherry and Sikkim
— are yet to adopt it. So far, 27 states UTs and 37 central ministries and departments have adopted gender
budgeting.
Interestingly, Odisha had a headstart as it adopted gender budgeting in 2004-05.
Source: https://economictimes.indiatimes.com/news/economy/policy/view-assessing-indias-gender-
budget/articleshow/80906668.cmsv
https://timesofindia.indiatimes.com/india/nine-states-uts-yet-to-adopt-gender-
budgeting/articleshow/81761571.cms

Q.44) Consider the following statements:


1. In a pro-cyclical fiscal policy, the government reinforces the business cycle.
2. The countercyclical fiscal policy usually causes deepening of the ongoing recession.
Which of the statements given above is/are correct?
a) 1 only
b) 2 only
c) Both 1 and 2
d) Neither 1 nor 2

Ans) a
Exp) Option a is correct.
Cyclicality of the fiscal policy simply refers to a change in direction of government expenditure and taxes based
on economic conditions. These pertain to decisions by policymakers based on the fluctuations in economic
growth. There are two types of cyclical fiscal policies - counter-cyclical and pro-cyclical.
Statement 1 is correct. In a pro-cyclical fiscal policy, the government reinforces the business cycle by being
expansionary (increasingg Govt. Expenditure or/and decreasing Taxes) during good times and contractionary
(decreasing Govt. Expenditure or /and increasing Taxes) during recessions. Pursuing a pro-cyclical fiscal policy
is generally regarded as dangerous. It could raise macroeconomic volatility, depress investment in real and
human capital, hamper growth and harm the poor. The outcome is deepening of recession and amplifying
expansions, thereby increasing fluctuations in the business cycle.
Statement 2 is incorrect. Counter-cyclical Fiscal Policy refers to the steps taken by the government that go
against the direction of the economic or business cycle. This means that during a recession or slowdown, the
government increases expenditure and reduces taxes to create a demand that can drive an economic boom. On
the other hand, during a boom in the economy, the policy aims at raising taxes and cutting public expenditure to
control inflation and debt. The outcome is softening of the recession and moderates the expansions, thereby
decreasing fluctuations in the business cycle.
Knowledge Base:

Forum Learning Centre: Delhi - 2nd Floor, IAPL House, 19 Pusa Road, Karol Bagh, New Delhi - 110005 | Patna - 2nd floor, AG Palace, E Boring Canal Road,
Patna, Bihar 800001 | Hyderabad - 1st & 2nd Floor, SM Plaza, RTC X Rd, Indira Park Road, Jawahar Nagar, Hyderabad, Telangana 500020
9821711605 | https://academy.forumias.com | admissions@forumias.academy | helpdesk@forumias.academy
Page 27 of 31

SFG 2022 | LEVEL 1 | Test #12 – Solutions |


Counter-cyclical fiscal policy works through multiple channels.
One, an expansion in government expenditure cushions the contraction in output by offsetting the decline in
consumption and investment.
Two, higher government spending builds confidence in tough times. Through this policy, governments are able
to show their commitment to sound fiscal management. This in turn gives confidence to the private sector that
the economy will not fluctuate too much. It helps businessmen overcome risk aversion and brings animal spirits
in the economy.
Source: https://www.moneycontrol.com/news/business/economy/explained-what-is-counter-cyclical-
fiscal-policy-that-economic-survey-2021-refers-to-6418421.html
https://www.indiabudget.gov.in/economicsurvey/doc/vol1chapter/echap02_vol1.pdf

Q.45) Consider the following statements:


1. The quantity of imported edible oils is more than the domestic production of edible oils in the last five years.
2. The Government does not impose any customs duty on all the imported edible oils as a special case.
Which of the statements given above is/are correct?
a) 1 only
b) 2 only
c) Both 1 and 2
d) Neither 1 nor 2

Ans) a
Exp) Option a is correct.
Statement 1 is correct. India imports most of the edible oil it consumes, unlike most other agricultural products
which are produced locally. India has an aggregate demand of around 26 million tons of edible oils per year. In
2019, India imported around 15 million tons of edible oils worth approximately Rs 7,300 crore, which accounted
for 40 per cent of the agricultural imports bill and three per cent of the overall import bill of the country. The
rest of the demand of around 10 million tons was fulfilled by domestic production. The domestic production can
only meet about 30-40% per cent of the total demand for edible oils, necessitating its import.
Palm oil accounted for the lion’s share of the total imports (62 per cent), followed by soya oil and sunflower oil
(21 per cent and 16 per cent, respectively).
Statement 2 is incorrect. The Government does impose customs duty on the imported edible oils.
In order to harmonize the interests of farmers, processors and consumers and at the same time, regulate large
import of edible oils to the extent possible, import duty structure on edible oils is reviewed from time to time.
In the budget 2021-22, the Government slashed the basic import duty on crude palm oil (CPO) to 15% from 27.5%
earlier. The basic custom duty on soybean oil and sunflower oil also cut 15% from 35% earlier. The government
has also proposed 17.5% cess on CPO and 20% cess on crude soybean and sunflower oil.
Source) UPSC Prelims 2018

Q.46) If external debt vis-s-vis internal debt in government's total debt increases, what would be its likely impact
on Indian economy?
1. It may bring down the interest that the government pays on its debt.
2. In the short term, it may lead to the rupee depreciating against the dollar.
3. It may have negative impact on Make in India initiative.
4. It can hamper countries' ability to invest in social and physical infrastructure.
Select the correct code from the options given below:
a) 1 and 2 only
b) 1 and 3 only
c) 2 and 4 only
d)1, 3 and 4 only

Ans) d
Exp) Option d is correct.

Forum Learning Centre: Delhi - 2nd Floor, IAPL House, 19 Pusa Road, Karol Bagh, New Delhi - 110005 | Patna - 2nd floor, AG Palace, E Boring Canal Road,
Patna, Bihar 800001 | Hyderabad - 1st & 2nd Floor, SM Plaza, RTC X Rd, Indira Park Road, Jawahar Nagar, Hyderabad, Telangana 500020
9821711605 | https://academy.forumias.com | admissions@forumias.academy | helpdesk@forumias.academy
Page 28 of 31

SFG 2022 | LEVEL 1 | Test #12 – Solutions |


Sovereign borrowing means raising the money from foreign markets by issuing government bonds by the
government/central bank of a country.
Statement 1 is correct. Global interest rates are at historic lows, so the government should tap the potential and
allocate needed money for country’s growth and development. Borrowing in dollars is expected to be cheaper,
and hence, bring down the interest that the government pays on its debt.
Statement 2 is incorrect. It may lead to the rupee appreciating against the dollar, at least in the short-run. When
the bonds are sold and the dollars (or any other foreign currency for that matter) are brought back to India, they
will have to be converted into rupees. This will push up the demand for rupees and eventually lead to the rupee
appreciating in value against the dollar.
Statement 3 is correct. In the short-term, an appreciating rupee will hurt India's exports, which are struggling
already. It will make imports cheaper and hurt domestic producers competing against them. Government of India
had launched “Make in India” scheme to promote manufacturing in India. But with the appreciation of rupee,
imports will be cheaper and it may impact the make in India initiative.
Statement 4 is correct. Excessive levels of foreign debt can hamper countries' ability to invest in their economic
future—whether it be via infrastructure, education, or health care—as their limited revenue goes to servicing
their loans. This thwarts long-term economic growth.
The United Nations has also linked high levels of foreign debt and a government's dependency on foreign
assistance to human rights abuses. Economic distress causes governments to cut social spending, and reduces
the resources it has to enforce labour standards and human rights, the U.N. says.
Knowledge Base:
India’s overall savings rate has declined to 30% from 34.6% over five years, ending FY2016-17. The worst dip was
seen in the household sector, the largest contributor to savings in the economy, dropping to 16.3% from 23.6%
over the period. So, with decline in savings and costly external commercial borrowings, sovereign borrowings
look promising.
The theory is that by borrowing abroad, the government won't be putting pressure on Indian savings, like it has
in recent years. And there will be less crowding out of private companies looking to borrow and, in the process,
slightly lower interest rates.
Assuming the borrowing is in US dollars, the rupee is likely to depreciate against the dollar in the long-term given
that India's inflation is significantly higher than that of the US. Then the lower interest rate cost will be more than
made up for by the government needing more rupees to buy dollars to repay the loan as well as pay interest on
it.
Source: https://blog.forumias.com/7-pm-time-ripe-for-sovereign-external-borrowing-17th-july-2019/
https://www.bbc.com/news/world-asia-india-48960315
https://www.investopedia.com/terms/f/foreign-debt.asp

Q.47) The Government has recently increased the foreign direct investment (FDI) limit in the insurance sector.
Which of the following is/are the likely consequence(s) to Indian economy from this step?
1. Improvement in capital availability in insurance sector.
2. Cost of insurance products to consumers may reduce
3. Insurance companies in India can now be foreign-owned and foreign-controlled
4. Insurance penetration is likely to double in the next five years from the present level of 20% of GDP.
Select the correct answer using the code given below:
a) 1 and 2 only
b) 1 only
c) 1, 2 and 3 only
d) 2, 3 and 4 only

Ans) c
Exp) Option c is correct.
The Government has increased the foreign direct investment (FDI) limit in the insurance sector to 74% from 49%.
Statements 1, 2 and 3 are correct.
Raising the foreign investment limit (FDI) in the insurance sector may provide the following benefits:

Forum Learning Centre: Delhi - 2nd Floor, IAPL House, 19 Pusa Road, Karol Bagh, New Delhi - 110005 | Patna - 2nd floor, AG Palace, E Boring Canal Road,
Patna, Bihar 800001 | Hyderabad - 1st & 2nd Floor, SM Plaza, RTC X Rd, Indira Park Road, Jawahar Nagar, Hyderabad, Telangana 500020
9821711605 | https://academy.forumias.com | admissions@forumias.academy | helpdesk@forumias.academy
Page 29 of 31

SFG 2022 | LEVEL 1 | Test #12 – Solutions |


1) Improve capital availability in the insurance sector: A higher FDI limit will help insurance companies access
foreign capital to meet their growth requirements. Insurance is a capital-intensive business. Simply put, as an
insurance company sells more policies and collects premiums from policy holders, it needs higher capital to
ensure that it is able to meet the future claims.
2) Lower cost of insurance products: Higher FDI limits could see more global insurance firms and their best
practices entering India. This could mean higher competition and better pricing of insurance products. Policy
holders will get a wide choice, access to more innovative products and a better customer service and claims
settlement experience.
3) Foreign Ownership of Insurance companies: Foreign direct investment (FDI) limit in the insurance sector is
increased to 74% from 49%. Increase in the FDI cap means insurance companies can now be foreign-owned
and -controlled as against the current situation wherein they are only Indian-owned and -controlled.
Statement 4 is incorrect. Insurance penetration in India is currently at 3.7% (not 20%) of the gross domestic
product (GDP) compared to the world average of 6.31%. With increased FDI limit, insurance penetration is likely
to increase. However, it is not likely to double in the next five years.
Source: 74% FDI in insurance: What this means for policy holders, Indian companies & promoters (theprint.in)
Cabinet clears "74% FDI in Insurance Sector"|ForumIAS Blog
https://www.livemint.com/insurance/news/finance-ministry-notifies-rules-for-increased-fdi-in-the-
insurance-sector-11621528712819.html

Q.48) If there is a growth of revenue deficit as a percentage of fiscal deficit, what could this possibly indicate?
1. Creation of large number of new airports and roads by the Government.
2. Increase in the subsidies distributed in the country.
3. Lower net capital accumulation in the given period.
Select the correct option using the code given below:
a) 1 and 2 only
b) 2 and 3 only
c) 1 only
d) 1, 2 and 3

Ans) b
Exp) Option b is correct.
If the balance of total revenue receipts and total revenue expenditures turns out to be negative it is known as
revenue deficit.
The difference between total revenue and total expenditure of the government is termed as fiscal deficit. It is an
indication of the total borrowings needed by the government. While calculating the total revenue, borrowings
are not included.
Statement 1 is incorrect – As Fiscal Deficit = Revenue Deficit + Capital Expenditure – Capital Receipts excluding
borrowings, an increase in revenue deficit with fiscal deficit being constant will lead to decrease in capital
expenditure. This will lead to less assets creation by the government which might lead to less infrastructure
development.
Statement 2 is correct – Subsidies comes under the revenue expenditure of the government. It is a recurring
expenditure and forms the part of revenue deficit. So with increasing subsidies there will be increase in revenue
deficit.
Statement 3 is correct – As seen from first statement, there is less capital expenditure due to increasing revenue
deficit, the net capital accumulation will hence be lower in the given period. Net Capital accumulation is the
second part i.e., capital expenditure – capital receipts excluding borrowings which will reduce.
Source: https://economictimes.indiatimes.com/definition/fiscal-deficit
https://economictimes.indiatimes.com/budget-faqs/budget-2020-types-of-
deficits/articleshow/73159003.cms?from=mdr

Q.49) With reference to Credit to GDP Ratio, which of the following statements is/are correct?
1. India’s credit to GDP ratio is just half of the average of G20 countries.
2. The lower credit to GDP ratio shows the need of more formalization of credit in the market.

Forum Learning Centre: Delhi - 2nd Floor, IAPL House, 19 Pusa Road, Karol Bagh, New Delhi - 110005 | Patna - 2nd floor, AG Palace, E Boring Canal Road,
Patna, Bihar 800001 | Hyderabad - 1st & 2nd Floor, SM Plaza, RTC X Rd, Indira Park Road, Jawahar Nagar, Hyderabad, Telangana 500020
9821711605 | https://academy.forumias.com | admissions@forumias.academy | helpdesk@forumias.academy
Page 30 of 31

SFG 2022 | LEVEL 1 | Test #12 – Solutions |


3. A lower credit to GDP gap indicates resilience or the ability of the economy to payback debt.
Select the correct answer using the code given below:
a) 1 and 2 only
b) 1 and 3 only
c) 2 and 3 only
d) 1, 2 and 3

Ans) d
Exp) Option d is correct.
According to analysts, bank credit growth is a key indicator of economic growth and a credit-GDP ratio of 100
per cent is the ideal, which indicates robust demand for credit without the fear of a bubble in the making.
Statement 1 is correct – According to BIS data, although the credit to GDP ratio of India increased to five-year
high of 56% in 2020, it is way behind its peers and just half of the G20 average. It is still the second lowest among
all its Asian peers. When it comes to the emerging market peers, it is 135.5% and at 88.7% in advanced economies.
Statement 2 is correct - A higher credit-to-GDP ratio indicates aggressive and active participation of the banking
sector in the real economy. While a lower number shows the need for more formal credit. This is where the role
of bank comes, where they need to work towards more formalization of credit to people. Also, this is the major
argument for privatization of state-run banks.
Statement 3 is correct - The low credit-to-GDP ratio of India has ensured that the credit-to-GDP gap (which is
a measure of risks associated with higher lending to businesses and households over long-term) is at a negative
5.7%, which is among the lowest in Asia. This shows the ability of the economy to payback debt. Higher gaps show
trouble for the financial system, which as best exemplified in the 2008 sub-prime housing crisis in the US. Several
Asian economies such as Japan, Korea and Hong Kong have alarming gaps at 28%, 28% and 18%, respectively.
Knowledge Base:
The credit-to-GDP gap ("credit gap") is defined as the difference between the credit-to-GDP ratio and its long-
term trend. Credit-to-GDP ratio tends to rise during the period of economic boom and fall during the period of
economic downturn. The difference of the credit-to-GDP ratio from its long-term trend, viz., credit-to-GDP gap
(actual credit-to-GDP ratio less long term credit-to-GDP trend) indicates the build-up of excessive credit growth
in an economy and system-wide risk as a precursor to the crisis. The CCCB should, therefore, be build up when
the credit-to-GDP gap exceeds a defined threshold.
Source: https://www.livemint.com/news/india/indias-bank-credit-to-gdp-ratio-inches-up-to-56-in-2020-
but-still-way-behind-peers-bis-data-11624543454835.html
https://www.rbi.org.in/scripts/PublicationReportDetails.aspx?UrlPage=&ID=797

Q.50) With respect to the recommendations of the 15th Finance Commission, consider the following statements:
1. Cost sharing patterns, between Centre and states, for disaster management funds to be 90:10 for all states.
2. No grants will be released to local bodies if the state does not constitute State Finance Commission by March
2024.
3. It provides for sector specific grants for certain sectors including health and education.
Which of the above statements is/are correct?
a) 2 only
b) 1 and 3 only
c) 2 and 3 only
d) 1, 2 and 3

Ans) c
Exp) Option c is correct.
The Finance Commission is a constitutional body formed by the President of India to give suggestions on centre-
state financial relations. The 15th Finance Commission (Chair: Mr. N. K. Singh) was required to submit two
reports. The first report, consisting of recommendations for the financial year 2020-21, was tabled in Parliament
in February 2020. The final report with recommendations for the 2021-26 period was tabled in Parliament on
February 1, 2021.

Forum Learning Centre: Delhi - 2nd Floor, IAPL House, 19 Pusa Road, Karol Bagh, New Delhi - 110005 | Patna - 2nd floor, AG Palace, E Boring Canal Road,
Patna, Bihar 800001 | Hyderabad - 1st & 2nd Floor, SM Plaza, RTC X Rd, Indira Park Road, Jawahar Nagar, Hyderabad, Telangana 500020
9821711605 | https://academy.forumias.com | admissions@forumias.academy | helpdesk@forumias.academy
Page 31 of 31

SFG 2022 | LEVEL 1 | Test #12 – Solutions |


Statement 1 is incorrect - The Commission recommended retaining the existing cost-sharing patterns between
the centre and states for disaster management funds. The cost-sharing pattern between centre and states is: (i)
90:10 for north-eastern and Himalayan states, and (ii) 75:25 for all other states. State disaster management funds
will have a corpus of Rs 1.6 lakh crore (centre’s share is Rs 1.2 lakh crore).
Statement 2 is correct – The commission has recommended several conditions for availing of grants released to
local bodies (other than health grants). This includes publishing provisional and audited accounts in the public
domain and improvement in the collection of property taxes (an additional requirement after 2021-22 for urban
bodies). Also, an additional condition is the establishment of State Finance Commission by March 2024, post
which no grants will be released if not established.
Statement 3 is correct – Sector-specific grants of Rs 1.3 lakh crore will be given to states for eight sectors: (i)
health, (ii) school education, (iii) higher education, (iv) implementation of agricultural reforms, (v) maintenance
of PMGSY roads, (vi) judiciary, (vii) statistics, and (viii) aspirational districts and blocks. A portion of these grants
will be performance-linked.
Knowledge Base: Criteria for devolution of funds:
Population (15%),
Area (15%),
Forest and Ecology (10%),
Income Distance (45%),
Demographic performance (12.5%),
Tax Effort (2.5%)
Income distance: Income distance is the distance of a state’s income from the state with the highest
income. Income of a state has been computed as average per capita GSDP during the three-year period between
2016-17 and 2018-19. A state with lower per capita income will have a higher share to maintain equity among
states.
Demographic performance: The Terms of Reference of the Commission required it to use the population data of
2011 while making recommendations. Accordingly, the Commission used 2011 population data for its
recommendations. The demographic performance criterion has been used to reward efforts made by states in
controlling their population. States with a lower fertility ratio will be scored higher on this criterion.
Forest and ecology: This criterion has been arrived at by calculating the share of the dense forest of each state
in the total dense forest of all the states.
Tax and fiscal efforts: This criterion has been used to reward states with higher tax collection efficiency. It is
measured as the ratio of the average per capita own tax revenue and the average per capita state GDP during the
three years between 2016-17 and 2018-19.
Source: https://prsindia.org/policy/report-summaries/report-15th-finance-commission-2021-26

Forum Learning Centre: Delhi - 2nd Floor, IAPL House, 19 Pusa Road, Karol Bagh, New Delhi - 110005 | Patna - 2nd floor, AG Palace, E Boring Canal Road,
Patna, Bihar 800001 | Hyderabad - 1st & 2nd Floor, SM Plaza, RTC X Rd, Indira Park Road, Jawahar Nagar, Hyderabad, Telangana 500020
9821711605 | https://academy.forumias.com | admissions@forumias.academy | helpdesk@forumias.academy

You might also like